Difference between revisions of "2002 AMC 10P Problems/Problem 5"

(Problem 5)
Line 1: Line 1:
== Problem 5 ==
+
== Problem ==
 
Let <math>(a_n)_{n \geq 1}</math> be a sequence such that <math>a_1 = 1</math> and <math>3a_{n+1} - 3a_n = 1</math> for all <math>n \geq 1.</math> Find <math>a_{2002}.</math>
 
Let <math>(a_n)_{n \geq 1}</math> be a sequence such that <math>a_1 = 1</math> and <math>3a_{n+1} - 3a_n = 1</math> for all <math>n \geq 1.</math> Find <math>a_{2002}.</math>
  

Revision as of 18:44, 14 July 2024

Problem

Let $(a_n)_{n \geq 1}$ be a sequence such that $a_1 = 1$ and $3a_{n+1} - 3a_n = 1$ for all $n \geq 1.$ Find $a_{2002}.$

$\text{(A) }666 \qquad \text{(B) }667 \qquad \text{(C) }668 \qquad \text{(D) }669 \qquad \text{(E) }670$

Solution 1

See also

2002 AMC 10P (ProblemsAnswer KeyResources)
Preceded by
Problem 4
Followed by
Problem 6
1 2 3 4 5 6 7 8 9 10 11 12 13 14 15 16 17 18 19 20 21 22 23 24 25
All AMC 10 Problems and Solutions

The problems on this page are copyrighted by the Mathematical Association of America's American Mathematics Competitions. AMC logo.png